Investments Flashcards

You may prefer our related Brainscape-certified flashcards:
1
Q

Concerning IPOs and the agreement between the issuer and the investment banking firm:

What is a “best efforts agreement” and a “firm commitment agreement”?

A

Best efforts: The investment bank sells the securities to investors and unsold shares are returned to the issuer (company selling their shares).

Firm commitment: The investment bank purchases all the securities from the firm and then sells the securities to the public.

How well did you know this?
1
Not at all
2
3
4
5
Perfectly
2
Q

What is The Efficient Market Hypothesis?

Who coined it?

A

Financial markets are informationally efficient (prices reflect relevant information).

Investors form rational expectations regarding future price movements.

Security prices follow a random walk (price changes are random and unpredictable).

Changes in relevant information (which are random) will be instantaneously reflected in changes in price.

Price changes are virtually impossible to predict.

Coined by Eugene Fama.

How well did you know this?
1
Not at all
2
3
4
5
Perfectly
3
Q

What are the three levels of efficiency and what do they reflect?

A
  1. Weak Form Efficiency:
    Asset prices reflect historical pricing and volume information.
    Investors can beat the market with fundamental analysis and insider trading.
    Technical analysis is useless
  2. Semi-Strong Form:
    Asset prices reflect all publicly available information.
    Suggests technical and fundamental analysis are useless - investors can only beat the market with insider trading.
  3. Strong form:
    Asset prices reflect all relevant information, including private information.
    Suggests all attempts to beat the market are useless - even with insider trading.
How well did you know this?
1
Not at all
2
3
4
5
Perfectly
4
Q

What is the mark where an Investment Advisor must register with the SEC as opposed to their respective state regulator?

A

$100mm AUM

How well did you know this?
1
Not at all
2
3
4
5
Perfectly
5
Q

What is Holding Period Return (HPR)?

What is the equation for it?

A

HPR is a measure of return that includes Capital Appreciation or loss and Current Income.

It does not take into account the time it took to get the return.

How well did you know this?
1
Not at all
2
3
4
5
Perfectly
6
Q

What is the annualized return formula?

A

(1 + Rp)^N - 1

Rp = return for the period being measured
N = number of periods in a year

Ex. Assume Jack earned a quarterly return of 3%. The annualized rate of return would be: (1.03)4 – 1 = 0.1255 = 12.55%

How well did you know this?
1
Not at all
2
3
4
5
Perfectly
7
Q

What is the Arithmetic Mean (AMR) and why is it not very accurate?

A

Arithmetic Mean is the sum or annual returns divided by the number of years - a simple average.

It doesn’t take compounding into account. This causes the AMR to be higher than actual returns when there are periods of negative returns.

How well did you know this?
1
Not at all
2
3
4
5
Perfectly
8
Q

What is the Geometric Mean (GMR)?

A

Calculates the compound annual return, assuming all earnings remain invested. The formula is as follows:

n = number of returns
rn = actual return for period n

How well did you know this?
1
Not at all
2
3
4
5
Perfectly
9
Q

What is Internal Rate of Return (IRR) and how is it calculated?

A

The internal rate of return (IRR) is the earnings rate for a series of cash inflows and outflows over a period of time while assuming all earnings are reinvested.

You do not need to complete the IRR calculation by hand. Review the math problem for a walk through on how to do it on your calculator.

How well did you know this?
1
Not at all
2
3
4
5
Perfectly
10
Q

What is Dollar-Weighted Return?

A

Measures the effect of all of the cash flows an investor controls. It combines the timing and dollar volume of investor trades during a period as well as the performance of the investment security.

Used for assessing the performance of the person who controls the cash flows.

Also known as internal rate of internal rate of return (IRR).

How well did you know this?
1
Not at all
2
3
4
5
Perfectly
11
Q

What is Time-Weighted Return?

A

Measures the effect of cash flows associated with an investment. It ignores the dollar volume and timing of investor-driven trades.

It assumes a buy-and-hold approach.

Mutual funds report their returns using a time-weighted return.

Used for assessing the performance of the investment itself (or, of an investment manager who does not control the cash flows).

Also known as geometric mean return (GMR).

How well did you know this?
1
Not at all
2
3
4
5
Perfectly
12
Q

If an investor times the market well, which will be higher - Dollar-Weighted or Time-Weighted return?

A

Dollar-Weighted

How well did you know this?
1
Not at all
2
3
4
5
Perfectly
13
Q

What is systematic risk and what are 5 examples?

A

Non-Diversifiable Risk “Inherent in the System”

  1. Purchasing Power (inflation reduces the value of returns)
  2. Reinvestment Rate (fall in interest rates affects assets that mature in future)
  3. Interest Rate (rise in interest rates reduces the value of existing income assets)
  4. Market Risk (overall decline in the stock market)
  5. Exchange Rate (investments in foreign assets decline due to currency values)
How well did you know this?
1
Not at all
2
3
4
5
Perfectly
14
Q

What is unsystematic-risk and what are 3 examples?

A

Unsystematic risk can be diversified away by combining multiple asset classes and industries in a portfolio.

Business
The riskiness of a specific business (operations, management style, earnings variability).

Country
Uncertainties due to international political and economic risks.

Financial
The capital structure of the firm (how much debt the firm uses).

How well did you know this?
1
Not at all
2
3
4
5
Perfectly
15
Q

If you want to reduce risk, you want to INCREASE or DECREASE the standard deviation?

A

Decrease

How well did you know this?
1
Not at all
2
3
4
5
Perfectly
16
Q

How do you calculate Standard Deviation?

A
  1. Estimate the arithmetic mean of the returns of the portfolio.
  2. Subtract the mean from each year’s return.
  3. Square each of those differences (“deviations”) to ensure that all numbers are positive.
  4. Add up all the squared numbers and divide by (n - 1) (number of returns - 1)
How well did you know this?
1
Not at all
2
3
4
5
Perfectly
17
Q

A Bell Curve skewed to the LEFT has NEGATIVE or POSITIVE skewness?

If a bell curve is skewed to the left, the hump will be on the RIGHT or the LEFT?

A

NEGATIVE

The hump will be on the RIGHT

How well did you know this?
1
Not at all
2
3
4
5
Perfectly
18
Q

What kind of stock are you picking if the stock has a high positive skewness?

A

A “lottery” type stock - low chance of success, high possible return.t

How well did you know this?
1
Not at all
2
3
4
5
Perfectly
19
Q

Beta measures:
I. Systematic Risk
II. Unsystematic risk

Is it I, II, neither or both?

A

I

Systematic risk only.

How well did you know this?
1
Not at all
2
3
4
5
Perfectly
20
Q

What is the “Coefficient of Determination”?

A

“R squared” (R2)
Indicates how much of the return on a security can be explained by return on he market.

R-squared is calculated by squaring the correlation coefficient:
Correlation = .8
R-squared = .8x.8 = 0.64 or 64%

How well did you know this?
1
Not at all
2
3
4
5
Perfectly
21
Q

By increasing the percentage of large-cap domestic stocks in her portfolio, an investment manager is making…

A

An asset allocation decision.

Asset allocation is an investment decision in which portfolio weights are assigned to reflect the investor’s risk profile.

How well did you know this?
1
Not at all
2
3
4
5
Perfectly
22
Q

Sharpe Ratio

A

Uses standard deviation (total risk) to estimate risk-adjusted return.

= (Portfolio Return – Risk-Free Rate) / (Standard Deviation)

How well did you know this?
1
Not at all
2
3
4
5
Perfectly
23
Q

Treynor Ratio

A

Uses systematic risk (beta) to adjust the return for risk.

= (fund return - risk free) / beta

24
Q

Jensen’s Alpha

A

Shows the difference bwtween actual and expected portfolio performance.

= portfolio return - [risk free + beta x (market portfolio return - risk free)]

25
Q

Explain callable bonds vs putable bonds.

A

Callable bonds allow the issuer to repay the bond prior to maturity. Good for issuers if interest rates go down. Bad for bond holders as they will lose the higher interest rate.

Puttable bonds allow the bond owner to sell the bond back to the issuer at a predetermined price. Good for the bond holder if interest rates go up - allows them to get the predetermined yield and reinvest the proceeds in a higher yield bond.

26
Q

Bond Swapping; Describe the following Bond Swaps:
1. Substitution Swap
2. Intermarket Spread Swap
3. Rate anticipation swaps

A
  1. Substitution Swap: Exchanging bonds with identical characteristics selling for different prices
  2. Intermarket Spread Swap: Exchanging similar bonds from two different market sectors. The goal is to capitalize on the spread between two similar bonds.
  3. Rate Anticipation Swap: Designed to take advantage of expected changes in interest rates.
27
Q

Bond Swapping; Describe the following Bond Swaps:
1. Pure Yield Pickup Swap
2. Tax Swap

A
  1. Pure Yield Pickup Swap: Designed to increase the yield through a swap, such as trading a lower-yielding bond for a higher yielding bond.
  2. Tax Swap: Motivated by making use of unrecognized capital losses. Be careful to avoid a wash sale.
28
Q

Describe:

Straight Voting
Cumulative voting
Proxy voting

When it comes to stock ownership

A

Straight Voting - Straight voting authorizes one vote per share of common stock and is the most common form of voting.

Cumulative voting - Cumulative voting allows one vote per share times the number of seats on the board of directors.

Proxy voting - Proxy voting involves sending a written authorization to an agent to cast the vote for the shareholder.

29
Q

The dividend-payout ratio is equal to

A) the dividend yield.
B) dividends per share divided by the par value of the stock per share.
C) dividends per share divided by the current market price per share.
D) dividends per share divided by earnings per share.

A

The correct answer is (D).
The dividend-payout ratio is equal to dividends per share divided by earnings per share.

30
Q

An order to buy or sell a certain quantity of a security at a specific or better price, but only after a specified price has been reached, is called a

A) stop-limit order.
B) stop-loss order.
C) stop order.
D) limit order.

A

The correct answer is (A).
A stop-limit order is an order to buy or sell a certain quantity at a specific or better price once a stop price has been reached.

31
Q

Describe the differences between forward and future contracts.

A

Futures contracts trade on an organized exchange and are standardized agreements.

Forward contracts are not standardized and are not traded on an exchange. They are private agreements between two parties.

32
Q

A futures contract:

A) is an agreement to buy or sell a specified amount of an asset at today’s price on the expiration date of the contract.
B) is a contract for a transaction at a specific date in the future by the buyer and the seller of the asset.
C) is an agreement to buy or sell a specified amount of an asset at a predetermined price on the expiration date of the contract.
D) gives the buyer the right, but not the obligation, to buy an asset prior to expiration of the contract.

A

The correct answer is (C).
A futures contract locks in the price of an asset to be bought and sold at some future date. Both the buyer and seller of the contract are committed at the inception of the contract.

33
Q

It is the responsibility of FINRA to educate investors

A

True

34
Q

It is the responsibility of FINRA to write and enforce rules governing the activities of all registered investment advisors

A

False. The SEC and state securities regulators are responsible for that.

35
Q

Which is reduced by diversification - systematic or unsystematic risk?

A

Unsystematic risk is reduced by diversification.

36
Q

Which of the following is accurate regarding the capital market line (CML)?

A) The portfolios on the new efficient frontier are some combination of the risk-free asset and the market portfolio.
B) The CML is a line that begins at the risk-free rate of return and crosses the efficient frontier at the market portfolio.
C) The CML represents the most efficient portfolios of individual stocks.
D) The slope of the CML is an investor’s indifference curve.

A

The correct answer is (A).

The CML is a combination of the risk-free return and the market portfolio. Option (B) is incorrect, as the CML does not cross the efficient portfolio, rather it is tangent to it. Option (C) is incorrect as the CML consists of the market portfolio and the Rf. Option (D) is incorrect as the slope is the Sharpe Ratio.

37
Q

Most variation in portfolio returns over an extended period of time is attributable to

A) security selection.
B) market timing.
C) asset allocation.
D) market cycles.

A

The correct answer is (C).
Over time, asset allocation represents the largest impact on performance for investors.

38
Q

Which of the following statements regarding collateralized mortgage obligations (CMOs) is correct?

A) All tranches receive principal payments throughout the term.
B) All tranches receive interest payments throughout the term.
C) Tranches with shorter maturities are not subject to prepayment risk.
D) Tranches with longer maturities are not subject to default risk.

A

The correct answer is (B).
While interest is paid to all tranches, principal is paid only to the first tranche until that tranche is retired, and then it is paid to the second tranche until it is retired, and so on. All tranches are subject to default risk and prepayment risk.

39
Q

A high-income investor owns a portfolio of bonds. Interest rates abruptly and sharply increased. Which of the following strategies would be most helpful in order to take advantage of the decrease in the value of this investor’s fixed-income securities?

A) The tax swap
B) The spread swap
C) The bullet strategy
D) The barbell strategy

A

The correct answer is (A).

As interest rates increase, the price of bonds decreases. This drop in value may create an opportunity to recognize the capital loss for tax purposes, which is the purpose of the tax swap.

40
Q

Parker, who lives in Covington, Louisiana, purchased three bonds from a company based in Brazil that were yielding 9.75 percent and paid a 12 percent coupon semi-annually. The company went bankrupt, and Parker never received his money. What type of risk was he subject to when he purchased the bond?

A) Interest-rate risk
B) Default risk
C) Exchange-rate risk
D) All of the above

A

The correct answer is (D).
The Brazilian bond is subject to exchange-rate risk, default risk, and interest-rate risk.

41
Q

Which of the following statements is/are correct?

I. Longer maturity bonds are less volatile than shorter term bonds when interest rates fluctuate.
II. Higher coupon bonds are less volatile than lower coupon bonds when interest rates fluctuate.

A) I only
B) II only
C) Both I and II
D) Neither I nor II

A

The correct answer is (B).
When interest rates change, bonds with higher coupon payments and shorter maturities are less volatile.

42
Q

Which of the following is an index designed to track the performance of large- and mid-cap securities from 21 developed countries?

A) Wilshire 21
B) MSCI EAFE
C) Barron’s 400
D) Euro 21

A

The correct answer is (B).
The MSCI EAFE index is often used as a benchmark for foreign asset managers.

43
Q

Juan bought XYZ Company stock at $40 per share. Today, the stock sells for $52 per share. Juan likes the long-term prospects for XYZ stock but wants some protection against price decreases. Which of the following orders is the best way for Juan to both participate in future price increases and ensure a minimum profit of $6 per share?

A) A limit order to sell at $46
B) A stop-limit order, stop price = $47, limit price = $46
C) A limit order to buy at $40
D) A stop-sell order, stop price = $46

A

The correct answer is (B).
A limit order will not work as it is a sell order at or above $46 that would be executed today. It does not provide the downside protection Juan is looking for. The stop-limit order will place a limit order with a price of $46 once the stock reaches $47. The stop-sell order results in a market order once the stock price reaches $46, which likely will result in the order being filled below $46.

44
Q

Which of the following statements regarding the primary market is (are) correct?

I. Investment bankers most commonly use best efforts to facilitate primary offerings.
II. Initial public offerings are a significant component of the primary market.

A) I only
B) II only
C) Both I and II
D) Neither I nor II

A

The correct answer is (B).
Statement I is incorrect because investment bankers most commonly use a firm commitment to facilitate a primary offering.

45
Q

Kay Fields compares the fees and expenses of several equity funds. Fields will likely have the most difficulty in evaluating

A) trading costs.
B) the 12b-1 fees.
C) the back-end loads.
D) the front-end loads.

A

The correct answer is (A).
Trading costs are the most difficult for an investor to assess, partly because the other fees and expenses are disclosed to investors in the prospectus, but also because some trading costs are implicit. Examples of implicit trading costs are bid-ask spreads, market-impact costs, and opportunity costs.

46
Q

Which of the following statements concerning a unit investment trust is correct?

A) It has an unmanaged portfolio.
B) It usually consists of common stocks.
C) It is most appropriate for young people.
D) It requires constant management attention

A

The correct answer is (A).
Option (B) is incorrect as UITs generally consist of bonds. Option (C) is incorrect as UITs are better for generating income, not the typical objective of young people. Option (D) is incorrect as the fund is generally passive.

47
Q

Which of the following regarding futures contracts is least accurate?

A) Futures contracts are less liquid than forward contracts.
B) Futures contracts are marked to market.
C) Futures contracts are traded on a regulated exchange.
D) Futures contracts allow more delivery options than forward contracts.

A

The correct answer is (A).
Futures contracts are more liquid than forward contracts, because they are traded on organized exchanges and are standardized. Forward contracts are customized to the parties involved in the transaction.

48
Q

A corn farmer who wants to hedge the price of corn should enter into what type of contract?

A) Buy a corn futures contract.
B) Sell a corn futures contract.
C) Long position in a corn call-option contract.
D) Long position in a commodity put-option index.

A

The correct answer is (B).
A corn farmer would enter into a short (sell) futures contract, because he wants to sell his corn.

49
Q

Which of the following statements concerning convertible bonds is (are) correct?

I. Convertible bonds provide an inexpensive way to buy stock.
II. Convertible bonds provide a lower yield than do straight bonds of comparable credit risk and maturity because of the attractiveness of the conversion feature.

A) I only
B) II only
C) Both I and II
D) Neither I nor II

A

The correct answer is (B).
Statement I is incorrect because convertible bonds normally trade at a premium-to-conversion value. Thus, buying a convertible bond and then converting the bond provides an investor with fewer shares of stock than if the investor had purchased the stock outright directly at the start.

50
Q

A venture capitalist provides EUR 25 million to a business that has developed an innovative fishing product. Distribution channels have not yet been identified, although some revenues have been generated. The VC has most likely entered the process at the

A) bridge stage.
B) initial public offering stage.
C) startup stage.
D) expansion stage.

A

The correct answer is (C).
The startup stage is characterized by businesses that have a product to sell but need help with marketing, packaging, and distribution. The VC frequently acts as an expert in this stage by providing advice and consulting services. The startup stage can produce some revenues, but the business lacks mass production capabilities.

51
Q

Which of the following is a violation of the wash-sale rule?

A) You sell 27 shares of an S&P 500 index fund at a loss. Then 15 days later, you buy 42 shares of a Russell 3000 index fund.
B) You have brokerage accounts at two firms. At the first firm, you buy 100 shares of PQR stock. Then 22 days later, you sell 100 shares of PQR stock, bought the prior year, for a loss in your account at the other firm.
C) Your spouse buys 200 shares of UK stock. Then 31 days later, you sell 100 shares of the same stock, bought 43 days earlier, at a slight loss.
D) You sell 100 shares of UK stock at a slight gain. Then 4 days later, your spouse buys 200 shares of the same stock.

A

The correct answer is (B).
Option (A) is incorrect because the two indices are sufficiently different. Option (C) is incorrect because the purchase is more than 30 days prior to the sale. Option (D) is incorrect because the stock is sold at a gain.

52
Q
A
53
Q

Gene and Jean are married taxpayers who have a high income and are in the top marginal tax bracket. They own the home they live in. The home has greatly appreciated in value since he bought it and they are concerned about the taxes they will owe if they sell it. Which of the following statements best describe the tax-savings opportunities available to them?

A) If they have lived in the home at least 2 of the past 5 years, they will not pay taxes on the first $500,000 of gain.
B) They can depreciate the value of the home, saving on taxes today; and, when they sell the home, they can recapture the depreciation at a lower tax rate than the top marginal rate.
C) If they perform a Section 1031 exchange, they may be able to defer or avoid taxation on the gain.
D) If they gift the home to a relative, that relative will receive a step-up-in-basis; the relative can then sell the home without recognizing any gain.

A

The correct answer is (A).
As a married couple, Gene and Jean can exclude the first $500,000 of gain from their taxable income (assuming they lived in the home at least 2 of the past 5 years). Statements (B) and (C) are incorrect because these rules apply to business property but not personal property. Statement (D) is incorrect because the step-up-in-basis rules apply to inherited property but not gifted property.

54
Q

Which of the following statements regarding the tax treatment of municipal bonds is correct?

A) The 3.8% Net Investment Income Tax (NIIT) applies to the coupon payments received from municipal bonds.
B) Public purpose municipal bonds are subject to the alternative minimum tax (AMT).
C) Gains from the sale of municipal bonds are tax-free at the federal level.
D) Coupon payments from municipal bonds are generally tax-free at the federal level.

A

The correct answer is (D).
Unlike corporate bonds, whose coupon payments are taxed as ordinary interest income, the coupon payments from municipal bonds are not subject to tax under the regular tax system and are not subject to the NIIT. If the taxpayer is subject to AMT, the coupon payments from private activity municipal bonds are taxable; however, the coupon payments from public purpose municipal bonds remain tax-free. Gains from the sale of municipal bonds are taxable gains.

55
Q

A writer of a put or call may have tax consequences in all the following situations EXCEPT

A) when the option premium is received.
B) when the option is exercised.
C) when the option expires.
D) when the option position is closed.

A

The correct answer is (A).
There are no tax consequences when an option is initially written. Tax consequences only arise when the option is exercised, when the option expires, or when the option position is closed with an offsetting trade.